Notices
 

Browse & Filter

For page specific messages
For page author info
Enter a comma separated list of user names.
2654 records found.
Operations
Selected 20 rows in this page.  
Title+Summary Name Datesort ascending

[QUE/ME-14002] ME-PROBLEM

Node id: 3988page

Question

Show that the principal moments of inertia of a linear chain consisting of two kinds of atoms \(a,b\), with origin chosen to coincide with the centre of mass, is given by \[I_1=I_2=\frac{1}{M}\sum_{i\ne j}m_im_j \,d_{ij}^2, \qquad I_3=0.\] where the summation includes each pair of \(i,j\) atoms once and \(d_{ij}\)is the distance between atoms in the pair and \(M\) is the total mass. Verify that this gives correct answer for a triatomic molecule.

Solution

Let the positions of the masses be \(x_i\) measured from some origin on the line joining the atoms. The moment of inertia about the origin, \(I_0\) is given by \[ I_0 = \sum_i m_i x_i^2.\] Letting \(I_\text{cm}\) denote the moment of inertia about the centre of mass. Using the parallel axes theorem we have \[I_0= I_\text{cm} + M X^2 , \] where \[ X = \frac{1}{M} \sum_i m_i x_i\] is the position of the centre of mass and \(M\) is the total mass. Therefore we get \begin{eqnarray} I_\text{cm} &=& I_0 - M X^2 = \frac{1}{M}\Big\{M \sum_i x_i^2 - \Big(\sum_im_ix_i\Big)^2\Big\} \\ &=& \frac{1}{M}\Big\{\Big(\sum_jm_j\Big) \sum_i (m_i x_i^2) - \Big(\sum_im_ix_i\Big)\Big(\sum_j m_j x_j\Big) \Big\}\\ &=& \frac{1}{M}\sum_{ij}\Big\{ m_jm_i (x_i^2 -x_ix_j) \Big\}\\ &=& \frac{1}{M}\sum_{i\ne j}\Big\{ m_jm_i (x_i^2 -x_ix_j) \Big\}\\ &=& \frac{1}{2M}\sum_{i\ne j}\Big\{ m_jm_i (x_i^2 -2x_ix_j + x_j)^2\Big\}\label{EQ01} \end{eqnarray} In the last step, for \(S_{ij}\) symmetric under exchange \(i\leftrightarrow j\), we have used \[\sum_{ij} S_{ij} T_{ij} =\frac{1}{2}\sum_{ij} S_{ij} \big(T_{ij} + T_{ji}\big)\] In the sum in \eqRef{EQ01}, each pair {ij} is counted twice, hence we get \begin{eqnarray} I_\text{cm} &=& \frac{1}{M}\sum_\text{pairs}\Big\{ m_jm_i (x_i^2 -2x_ix_j + x_j)^2\Big\}\\ &=& \frac{1}{M} \sum_\text{pairs} (x_i-x_j)^2. \end{eqnarray} where now each pair is counted once.

shivahcu's picture 22-02-07 21:02:46 n

[QUE/ME-14001] ME-PROBLEM

Node id: 3987page



Question

A point of rigid body is pivoted in way that the body can rotate freely about a horizontal axis passing thorough the point of suspension. Show that the frequency of small oscillations is given by \[\omega^2= \frac{Mgh}{Mh^2 + I_1n_1^2+I_2n_2^2+I_3n_3^2}\] where \(I_1, I_2, I_3\) are principal moments of inertia about the centre of mass and \(h\) is distance of centre of mass from the point of suspension. Also \(\hat{n}=(n_1,n_2,n_3)\) is unit vector along the axis of rotation and the components of \(\hat{n}\) are taken w.r.t. the principal axes relative to the centre of mass of the body.
%FigBelow{10,-47}{40}{0}{me-fig-14006}

Solution

This is a problem about rotation of a rigid body about a fixed axis. In this case the axis of rotation is horizontal line passes through the point of suspension. The equation of motion is obtained by equating the rate of change of angular momentum to the torque. The angular momentum is \(I\dot{\theta}\), where \(I\) is the moment of inertia about the axis of rotation. The torque is given by \(Mgh\sin\theta\) and is in the clockwise direction of decreasing \(\theta\). Therefore the equation of motion becomes \begin{equation} \dd[L]{t} = - Mgh \sin\theta\\ \end{equation} Substituting \(L=I\dot{\theta}\) and making small amplitude approximation we get \begin{eqnarray} I \ddot{\theta} + Mgh \theta=0\\ \therefore \qquad \qquad \ddot{\theta} + \frac{Mgh}{I} \theta=0. \end{eqnarray} Thus the angular frequency of oscillations is \(\omega^2=\dfrac{Mgh}{I}\). Using the parallel axes theorem the moment of inertia can be written as \begin{equation} I= I_0 + Mh^2 \end{equation} where \(I_0\) is the moment of inertia about a parallel axis passing through the centre of mass. In terms of the principal moments of inertia the value of \(I_0\) is given by \begin{equation} I =I_1n_1^2 + I_2n_2^2 + I_3n_3^2. \end{equation} Thus we get the final answer \begin{equation} \omega^2=\frac{Mgh}{I_0 + I_1n_1^2 + I_2n_2^2 + I_3n_3^2}. \end{equation}

shivahcu's picture 22-02-07 21:02:17 n

[QUE/ME-09003] ME-PROBLEM

Node id: 3968page
  1. Potential energy of the system is \( V_0 \Big(\dfrac{1}{r}\Big). \)
  2. Potential energy of the system is \( V(r) = V_0 \Big(\dfrac{r^2}{R^3}\Big).\)
  3. The force between the particles as function of separation is given by \( - V_0 \Big(\dfrac{R}{r^2} + \dfrac{15 R^3}{r^4} \Big).\)

Here \(r\) denotes the separation of the particles and \(V_0, R\) are some positive constants. Assuming that the angular momentum of the particle is \(L=\sqrt{mV_0} R\). Here \(V_0\) and \(R\) are some positive constants. For the three cases find if a stable circular orbits exist. If yes, find the radius of the circular orbit and corresponding value of energy in terms of $V_0$ and $R$.

shivahcu's picture 22-02-07 21:02:50 n

[QUE/ME-09002] ME-PROBLEM

Node id: 3967page
shivahcu's picture 22-02-07 21:02:41 n

[QUE/ME-09001] ME-PROBLEM

Node id: 3966page
shivahcu's picture 22-02-07 21:02:25 n

[QUE/ME-02019] ME-PROBLEM

Node id: 3949page

Show that an rotation by an infinitesimal angle \(\Delta \theta\) about an axis \(hat{n}\) is equivalent to successive rotations by infinitesimal angles \(\alpha, \beta, \gamma\) about the three coordinate axes. Keeping first order terms in \(\Delta \theta\), find expressions for the angles \(\alpha, \beta, \gamma\) in terms of components of \(hat{n}\) and \(\Delta\theta\).

shivahcu's picture 22-02-07 21:02:23 n

[QUE/ME-02023] ME-PROBLEM

Node id: 3953page


Question

Find rotation matrix for a rotation by an angle \(\alpha\) about the axis \(1,2,1\) where \(\cos\alpha=\frac{3}{5}, \sin\alpha =\frac{4}{5}\).

Solution

The unit vector along the direction \((1,2,1)\) is given by \(\hat{n}=\frac{1}{\sqrt{6}}(1,2,1)\).\\ Under a rotation by an angle \(\alpha\) about axis \(\hat{n}=(n_1,n_2,n_3)\), the new components \(\vec{X}\) are related to old components \(\vec{x}\) by equation \begin{equation} \vec{X} = \vec{x} -\sin \alpha (\hat{n}\times\vec{x}) + (1-\cos\alpha)\hat{n}\times(\hat{n}\times\vec{x})) \end{equation} We compute \[\hat{n}\times\vec{x}=\big({n_2} {x_3}-{n_3} {x_2},{n_3} {x_1}-{n_1} {x_3},{n_1} {x_2}-{n_2} {x_1}\big)\] \begin{eqnarray} (\hat{n}\times(\hat{n}\times\vec{x}))_1&=&{n_2} ({n_1} {x_2}-{n_2} {x_1})-{n_3} ({n_3} {x_1}-{n_1} {x_3})\\ (\hat{n}\times(\hat{n}\times\vec{x}))_2&=&{n_3} ({n_2} {x_3}-{n_3} {x_2})-{n_1} ({n_1} {x_2}-{n_2} {x_1})\\ (\hat{n}\times(\hat{n}\times\vec{x}))_3&=& {n_1} ({n_3} {x_1}-{n_1} {x_3})-{n_2} ({n_2} {x_3}-{n_3} {x_2}) \end{eqnarray} Therefore \begin{eqnarray}\nonumber X_1&=&(1-\cos \alpha ) ({n_2} ({n_1} {x_2}-{n_2} {x_1})-{n_3} ({n_3} {x_1}-{n_1} {x_3}))-\sin \alpha ({n_2} {x_3}-{n_3} {x_2})+{x_1}\\\nonumber X_2&=&(1-\cos \alpha ) ({n_3} ({n_2} {x_3}-{n_3} {x_2})-{n_1} ({n_1} {x_2}-{n_2} {x_1}))-\sin \alpha ({n_3} {x_1}-{n_1} {x_3})+{x_2}\\\nonumber X_3&=& (1-\cos \alpha ) ({n_1} ({n_3} {x_1}-{n_1} {x_3})-{n_2} ({n_2} {x_3}-{n_3} {x_2}))-\sin \alpha ({n_1} {x_2}-{n_2} {x_1})+{x_3} \end{eqnarray} Therefore \begin{equation} \begin{pmatrix} X_1\\X_2\\X_3 \end{pmatrix} = \underline{\Rbb} \begin{pmatrix} x_1\\x_2\\x_3 \end{pmatrix}. \end{equation} where the matrix \(\Rbb\) is given by \begin{eqnarray}\nonumber \begin{pmatrix} -(1-\cos\alpha)(n_2^2+n_3^2) + 1 & (1-\cos\alpha)n_1n_2 + \sin\alpha n_3 & (1-\cos\alpha)n_3n_1 - \sin\alpha n_2\\ (1-\cos\alpha)n_1 n_2 -n_3\sin\alpha & -(1-\cos\alpha)(n_3^2+n_1^2) +1 & (1-\cos\alpha)n_2n_3 + n_1\sin\alpha\\ (1-\cos \alpha)n_1n_3 + n_2 \sin\alpha & (1-\cos\alpha)n_2n_3 -n_1 \sin\alpha & -(1-\cos\alpha)(n_1^2+n_2^2) + 1 \end{pmatrix} \end{eqnarray} Substituting values \begin{equation*} \hat{n}=\frac{1}{\sqrt{6}}(1,2,1),\quad (1-\cos\alpha)=\frac{2}{5}, \quad\sin\alpha=\frac{4}{5} \end{equation*} and simplifying gives % \begin{equation} \left(\begin{array}{ccc} \frac{2}{3} & \frac{2}{15} \left(1+\sqrt{6}\right) & \frac{1}{15} \left(1-4 \sqrt{6}\right) \\[2mm] \frac{2}{15} \left(1-\sqrt{6}\right) & \frac{13}{15} & \frac{2}{15} \left(1+\sqrt{6}\right) \\[2mm] \frac{1}{15} \left(1+4 \sqrt{6}\right) & \frac{2}{15} \left(1-\sqrt{6}\right) & \frac{2}{3} \end{array} \right) \end{equation}

 

shivahcu's picture 22-02-07 21:02:41 n

[QUE/ME-02022] ME-PROBLEM

Node id: 3952page

I Question 
Let \(S_{ij}\), \(A_{ij}\) and be symmetric, ansymmetric tensors respectively. Then for arbitrary tensor \(T_{ij}\), {\bf prove any one} of the following identities. \[(a)\ \ S_{ij}T_{ij}=\frac{1}{2}S_{ij}\big(T_{ij}+T_{ji}\big); \quad \quad (b)\ \ A_{ij}T_{ij}=\frac{1}{2}A_{ij}\big(T_{ij}-T_{ji}\big).\]
% \FigBelow{10,-25}{40}{0}{}{}\\[3mm]


II Solution 



Solution1

Consider the right hand side of the identity (a) to be proved and write all terms in the sum. There will be four terms as \(i\) and \(j\) take values 1 and 2. This gives \begin{eqnarray} \frac{1}{2}S_{ij}\big(T_{ij}+T_{ji}\big) &=&\frac{1}{2}\sum_{i=1}^2\sum_{j=1}^2S_{ij}\big(T_{ij}+T_{ji}\big)\\ &=& \frac{1}{2}\sum_{i=1}^2\Big( S_{i1}\big(T_{i1}+T_{1i}\big)+ S_{i2}\big(T_{i2}+T_{2i}\big)\Big)\\ &=&\frac{1}{2} \Big( S_{11}\big(T_{11}+T_{11}\big)+ S_{12}\big(T_{12}+T_{21}\big)\Big)\\ && +\frac{1}{2} \Big( S_{21}\big(T_{21}+T_{12}\big)+ S_{22}\big(T_{22}+T_{22}\big)\Big)\\ &=& S_{11}T_{11} +S_{12}T_{12} + S_{21}T_{21}+ S_{22}T_{22}\\ &=& \sum_{i=1}^2 \sum_{j=1}^2 S_{ij}T_{ij} = S_{ij}T_{ij} \end{eqnarray} Note:- This approach will not be useful for more complicated expressions.

Solution2

\begin{eqnarray} S_{ij}T_{ij} &=& S_{ji}T_{ij} \qquad \HighLight{Used symmetry property of S} \\ &=&S_{nm}T_{mn} \qquad \HighLight{renamed dummy indices}\\ &=& S_{ij}T_{ji} \qquad \HighLight{renamed dummy indices again} \label{EQ02} \end{eqnarray} Therefore the right hand side of (a) becomes \begin{eqnarray} \frac{1}{2}S_{ij}\big(T_{ij}+T_{ji}\big) &=&\frac{1}{2}S_{ij}T_{ij} +\frac{1}{2} S_{ij}T_{ji} \qquad \text{\HighLight{Now use \eqRef{EQ02} in second term}}\nonumber\\ &=& \frac{1}{2}S_{ij}T_{ij} +\frac{1}{2} S_{ij}T_{ij} = S_{ij}T_{ij}. \end{eqnarray}

Cutest Solution : 

\begin{eqnarray}\nonumber S_{ij}T_{ij} &=& S_{ji}T_{ji} \qquad \HighLight{Change Dummy indices}\\\nonumber S_{ij}T_{ij} &=& S_{ji}T_{ji} \qquad \HighLight{Use symmetry of S}\\ \text{add} \qquad S_{ij}T_{ij} &=& \frac{1}{2}S_{ij}\big(T_{ij}+T_{ji}\big)\nonumber \end{eqnarray}

shivahcu's picture 22-02-07 21:02:27 n

[QUE/ME-02021] ME-PROBLEM

Node id: 3951page

Question

A rotation takes a vector by an angle \(\alpha\) about axis (2,1,2) takes vector \(\underline{\sf A}\) to new vector \(\underline{\sf A}{'}\). Taking \(\alpha =\cos^{-1}\Big(\frac{3}{5}\Big), 0 < \alpha < \pi/2 \), find the rotation matrix \(R\), such that \(\underline{\sf B}=R \underline{\sf A}\) that relates two vectors.

Solution

shivahcu's picture 22-02-07 21:02:17 n

[QUE/ME-02020] ME-PROBLEM

Node id: 3950page

Show that the trace of rotation matrix for rotation by an angle \(\phi\)is given by \((1+\cos2\phi)\).

shivahcu's picture 22-02-07 19:02:38 n

[QUE/ME-02018] ME-PROBLEM

Node id: 3948page

A particle of mass m moves on the inner surface of a cone of revolution, whose semi-vertical angle is \(\alpha\), under the action of arepulsive force \({m\mu/r^3}\) from the axis; the angular momentum of the particle about the axis being \(m\surd \mu \tan \alpha\); prove that its path is an arc of a hyperbola whose eccentricity is \(\sec \alpha\). . [Math. Tripos, 1897.]

shivahcu's picture 22-02-07 19:02:57 n

[QUE/ME-02017] ME-PROBLEM

Node id: 3947page

Question 

 You are given two unit vectors \(\vec{m}=(1,1,1)\) and \(\vec{n}=(1,0,1)\). What should be the unit vector \(\hat{\ell}\) along an axis what should be the angle of rotation \(\alpha\) that will rotate vector \(\vec{n}\) and align it along the vector \(\vec{m}\).

Answer

\(\cos\alpha=\frac{\vec{m}\cdot\vec{n}}{|\vec{m}||\vec{n}|}=\frac{1}{\sqrt{3}} \)

Solution

Angle \(\alpha\) is given by \(\cos\alpha=\dfrac{\vec{m}\cdot\vec{n}}{|\vec{m}||\vec{n}|}=\dfrac{1}{\sqrt{3}} \);Clockwise rotation by \(\alpha \) about  axis along \(\hat{\ell}=\dfrac{\vec{m}\times\vec{n}}{|\vec{m}||\vec{n}|}= \frac{1}{\sqrt{2}}(1,0,-1)\);

shivahcu's picture 22-02-07 19:02:35 n

[QUE/ME-02016] ME-PROBLEM

Node id: 3946page

Question 

You are given two unit vectors \(\hat{\ell}\) and \(\hat{n}\) and angles \(\alpha,\gamma\). Find the axis \(\hat{m}\) and angle of rotation \(\beta\) such that a rotation \(R_{\hat{\ell}}(\alpha)\) followed by \(R_{\hat{m}}(\beta)\) and then by \(R_{\hat{n}}(\gamma)\) amounts to no rotation of an arbitrary vector.

Answer

Angle \(\cos\alpha=\hat{m}\cdot\hat{n}\) Clockwise rotation by \(\alpha \) about axis along \(\hat{\ell}=\hat{m}\times\hat{n}\);

shivahcu's picture 22-02-07 19:02:36 n

[QUE/ME-02015] ME-PROBLEM

Node id: 3945page

Show that \begin{equation*} [\vec{A},\vec{B},\vec{C}]^2 = \begin{vmatrix} \vec{A}.\vec{A} & \vec{A}.\vec{B} & \vec{A}.\vec{C} \\ \vec{B}.\vec{A} & \vec{B}.\vec{B} & \vec{B}.\vec{C} \\ \vec{C}.\vec{A} & \vec{C}.\vec{B} & \vec{C}.\vec{C} \end{vmatrix} \end{equation*} where \([\vec{A},\vec{B},\vec{C}]=\vec{A} . (\vec{B} \times \vec{C} ). \)

shivahcu's picture 22-02-07 19:02:16 n

[QUE/ME-02014] ME-PROBLEM

Node id: 3944page
  • Given a rotation matrix what property will you use to find the axis of rotation and the angle of rotation.
  • Verify that the matrix \begin{equation} R = \frac{1}{3}\begin{pmatrix} 2 & 2 & -1\\ -1& 2 & 2 \\2 & -1 & 2 \end{pmatrix} \end{equation} represents a proper rotation.
  • Find the axis and angle of rotation corresponding to the rotation matrix in (b).
shivahcu's picture 22-02-07 19:02:25 n

[QUE/ME-02013] ME-PROBLEM

Node id: 3943page

Write \(3\times 3\) rotation matrix corresponding to a rotation about axis \(\hat{n}\) by an angle \(\theta\).\hfill \GetQSource using the result \begin{equation}\label{EQ14} \vec{x}{'} = (\vec{x}\cdot\hat{n})\hat{n} - (\hat{n}\times\vec{x})\sin\theta - \hat{n}\times(\hat{n}\times\vec{x})\cos\theta. \end{equation} on the rotations by angle \(theta\) about axis \(hat{n}\) and

shivahcu's picture 22-02-07 19:02:04 n

[QUE/ME-02012] ME-PROBLEM

Node id: 3942page

Show that at least one eigenvalue of every rotation matrix is real. What are the ways in which all the three eigenvalues can be real? If the matrix has complex eigenvalues, show that they must come in complex conjugate pairs

shivahcu's picture 22-02-07 19:02:54 n

[QUE/ME-02011] ME-PROBLEM

Node id: 3941page

Using definition of Levi-Civita symbol, prove the following identities. \begin{eqnarray} \epsilon_{i\,j\,k}\,\epsilon_{i\,j\,k} \ &=&\ 6 \\ \epsilon_{i\,j\,k}\,\epsilon_{l\,j\,k} \ &=&\ 2\,\delta_{il} \\ \epsilon_{i\,j\,k}\,\epsilon_{l\,m\,k}\ &=&\ (\,\delta_{il}\,\delta_{jm}\,-\,\delta_{im}\,\delta_{jl}) \\ \epsilon_{i\,j\,k}\,\epsilon_{l\,m\,n}\ &=&\ \left(\begin{array}{ccc} \delta_{il}&\delta_{im}&\delta_{in}\\ \delta_{jl}&\delta_{jm}&\delta_{jn}\\ \delta_{kl}&\delta_{km}&\delta_{kn} \end{array} \right) \end{eqnarray} Use result on \(\vec{A}\times(\vec{B}\times{\vec{C})}\) to derive identity (3) of Q[1].

shivahcu's picture 22-02-07 19:02:18 n

[QUE/ME-02010] ME-PROBLEM

Node id: 3940page

Using definition of Levi-Civita epsilon symbol and Kronecker delta symbol to show that \begin{equation}\epsilon_{i\,j\,k}\,\epsilon_{k\,l\,m}\ =\ (\,\delta_{il}\,\delta_{jm}\,-\,\delta_{im}\,\delta_{jl}) \end{equation} Use this identity to prove that \begin{equation}\vec{A}\times(\vec{B}\times\vec{C})=(\vec{A}\cdot\vec{C})\vec{B} -(\vec { A } \cdot\vec{B})\vec{C}.\end{equation}

shivahcu's picture 22-02-07 19:02:21 n

[QUE/ME-02009] ME-PROBLEM

Node id: 3939page


Read the following theorem of Rodrigues and Hamilton
taken from Whittaker.
The theorem of Rodrigues and Hamilton.
Any two successive rotations about a fixed point can be compounded intoa single
rotation by means of a theorem, which may be stated as follows:Successive
rotations about three concurrent lines fixed in space, through twicethe angles
of the planes formed by them, restore a body to its original position. For let
the lines be denoted by OP, OQ, OR. Draw. Op, Oq, Orperpendicular to the planes
QOR, ROP, POQ respectively. Then if a body isrotated through two right angles
about Oq, and afterwards through two rightangles about Or, the position of OP is
on the whole unaffected, while Oq ismoved to the position occupied by its image
in the line Or; the effect is therefore the same as that of a rotation round OP
through twice the anglebetween the planes PR and PQ, which we may call the angle
RPQ. It follows that successive rotations round OP, OQ, OR through twice the
angles RPQ, PQR, QRP, respectively, are equivalent to successive rotations
through two right angles about the lines Oq, Or, Or, Op, Op, Oq; but the
latter rotations will clearly on the whole produce no displacement; which
establishes the theorem. 


Now solve the following problem. Following two rotations are preformed in
succession
(i) rotation by angle \(\alpha\) about axis \(\hat{n}\);
(ii) rotation by angle \(\beta\) about axis \(\hat{m}\).
Find the angle and axis of rotation that will produce the same result as the
combined effect of above two rotations.

shivahcu's picture 22-02-07 19:02:55 n

Pages

 
X